Jump to content

Drake Glau

Recommended Posts

Please help me with this question, I've been stuck for ages.

'Imagine an astronaut 2m outside the exterior walls of the Space shuttle, and 10m from the centre of mass of the Space shuttle. By making appropriate assumptions and approximations, calculate how long it would take for this astronaut to be pulled back to the Space shuttle by the force of gravity alone.'

The mass of the Space Shuttle is: 6.8 x 10^4 kg

Thanks in advance

Link to post
Share on other sites

  • 2 weeks later...
  • 3 weeks later...

Please help me with this question, I've been stuck for ages.

'Imagine an astronaut 2m outside the exterior walls of the Space shuttle, and 10m from the centre of mass of the Space shuttle. By making appropriate assumptions and approximations, calculate how long it would take for this astronaut to be pulled back to the Space shuttle by the force of gravity alone.'

The mass of the Space Shuttle is: 6.8 x 10^4 kg

Thanks in advance

http://physics.stackexchange.com/questions/19813/calculating-gravity-when-taking-into-account-the-change-of-gravitational-force

I thought about it for a while, and then I found this. Take a look at some of the stuff and their links to other works on the same topic. If it doesn't make sense, tell me.

On another note, can anyone explain the Lie Group SU(3) and how that relates to Quantum Chromodynamics? If you could give a simple analogy, that would be great.

Link to post
Share on other sites

  • 2 weeks later...
  • 4 months later...

Question: I don't understand why D is the answer - anyone help please :)



A cell of emf ε and internal resistance r delivers current to a small electric motor.


motor. 450 C of charge flows through the motor and 9000 J of energy are converted in the motor. 1800 J are dissipated in the cell. The emf of the cell is



A. 4.0V. B. 16V. C. 20V. D. 24V.


Link to post
Share on other sites

  • 1 month later...

<p>

You walk up a 1000m high mountain, which takes you 5 hours. Walking on the flat consumes about 400W. You are hoping to lose some of your 70kg mass with all this exercise but on the way you eat 4 small chocolate bars, which according to the wrapper will each supply 1130KJ

A) Calculate the energy you expend during the 5 hour walk. While you're climbing your body only works at a 15% efficiency.

  • I got 48 MJ as the total
B) Calculate the energy that must be obtained from your body after the energy of the chocolate has been used.
  • I got 4520KJ (but i'm not sure whether to account for the efficiency)
C) If fat converts to energy at a rate of 38MJkg-1, how much mass are you likely to lose?
400W, 15%=60W

5*60*60=18000

A watt is J/s so in 5 hours you would use 60J*18000s=1,080,000J or 1.08MJ

So for the walk you need 1.08MJ, the bars will provide 4*1130KJ which is 4520KJ (4.52MJ) but you will only use 15% of that so the bars provided an actualy energy output of 0.678MJ which means your body will provide 1.08-0.678=0.402MJ

(If you didn't account for efficient in this part you would have gotten a negative number http://www.ibsurvival.com/public/style_emoticons/#EMO_DIR#/pinch.gif)

0.402/38=0.0106Kg.

Surely, mgh states 70*9.81*1000 is the energy needed to get to the top.of the hill. At 15% efficiency that would mean required energy is

Person Energy = 70*9.81*1000/15%

The residual after chocolate is

Residual Energy = 4*1130,000 - Person Energy

Then take Residual and figure our how many kg are lost due to the Residual being negative, as in the chocolate was less than that used up the hill.

Not sure why I need to know flat walking value 400W..

Link to post
Share on other sites

  • 1 month later...

Hi guys I need some help with an uncertainty question.

If I wanted to calculate the weight of an object, I would use F=mg. I know 'g' and the mass is written on the object (let's say 100g). So F=(0.1)(9.81). But what are the uncertainties in this? Since the mass is written on the object, I did not have to weigh it on a weighing machine so I don't really see a source of uncertainty. So is the uncertainty 0 or should I just estimate an uncertainty?

Link to post
Share on other sites

Hi guys I need some help with an uncertainty question.

If I wanted to calculate the weight of an object, I would use F=mg. I know 'g' and the mass is written on the object (let's say 100g). So F=(0.1)(9.81). But what are the uncertainties in this? Since the mass is written on the object, I did not have to weigh it on a weighing machine so I don't really see a source of uncertainty. So is the uncertainty 0 or should I just estimate an uncertainty?

Is this for a lab? Assuming it is, and that mass is your IV then yeah, uncertainty would be zero and you can explain that by saying it's a standard mass or something of the sort - that's what our teacher told us to do.

Link to post
Share on other sites

  • 3 months later...

Hi guys! :)

Could anyone please explain electricity and magnetism in a nutshell to me, please? I realise that this is a very very broad topic, but any help at all would be so appreciated, particularly with the formulas involved, right-hand rule, and magnetic flux...

Sorry if someone has already asked any of this also!

This is such a broad topic that no one can really give a short summary on because there's so much, what in particular do you have questions about?

Link to post
Share on other sites

Hi guys! :)

Could anyone please explain electricity and magnetism in a nutshell to me, please? I realise that this is a very very broad topic, but any help at all would be so appreciated, particularly with the formulas involved, right-hand rule, and magnetic flux...

Sorry if someone has already asked any of this also!

This is such a broad topic that no one can really give a short summary on because there's so much, what in particular do you have questions about?

Yeah...Sorry about that, I should've been more specific! I just generally had a really tough time with electricity and magnetism as it was the first thing we learned for Physics HL, so if anyone knows any really great places to review/understand the concepts and any study guides that would be lovely! For particular questions I would say maybe how to find EMF and how it affects other variables, and the force exerted by current carrying wires. Those two I found particularly difficult!

Thank you so much and also I appreciate the "R U Mine", haha :)

Link to post
Share on other sites

Emf is the power supplied by the supply per unit current. That's the definition IB wants you to know. Although the underlined bit sounds redundant, in one of the past papers I did you actually had to state that in order to get any marks.

There are two main ways of finding emf. They are both in the data booklet. The first one is e=BLv

Where B is magnetic field strength

L is length on conductor

v is velocity of the conductor

The syllabus says that you need to be able to derive that one. Deriving questions rarely come up but you should probably know it just in case.

To understand the derivation you need to first know how emf is induced. Conductors (eg a piece of wire) contain free electrons. When a conductor moves through a magnetic field, the electrons experience a force. This force causes the electrons to move to one side of the conductor, and the atoms on the other side become positively charged. Now the conductor has a positive and a negative side which creates a potential difference (voltage) between the two sides. The electrons will stay where they are because the magnetic force and the electric force are at equilibrium.

So the derivation is:

F(B)=F(E)

qvB=Eq

E=vB

(delta)V=EL

(delta)V=vBL

e=vBL

q is charge, v is velocity, B is magnetic field strength, E is electric field strength, L is length, (delta)V is change in voltage and e is emf.

The other formula is based of Faraday's Law which says that 'An induced emf is proportional to the rate of change of the flux linkage'

e= -N(change in flux linkage / change in time) where N is the number of turns of wire.

I hope that kind of helps. Which other variables to you mean that are affected by emf?

Link to post
Share on other sites

  • 4 weeks later...

Hi guys,i need help on a question regarding thermal physics!

An ideal gas expands isothermally from a state X to a new state of volume V. The work done by the gas is W. Which of the following is correct for an adiabatic expansion of the gas from state X to a new state of volume V?

Change in internal energy - work done

A. Delta U>0 - greater than W

B. Delta U<0 - greater than W

C. Delta U>0 - less than W

D. Delta U<0 - less than W

Edited by Stefan
Link to post
Share on other sites

  • 2 months later...

My exams is in 2 days time

and im not even confident about paper 1 and paper 2 and i took physics HL

my school didnt have the proper teacher for nearly 1 semester which forced most of us to study by ourself

is there any suggestion of web/links that might help me?

for the few of mock exam conducted by my tuition course,i always get a pretty bad marks

Thanks!

Link to post
Share on other sites

  • 10 months later...

Hi guys,i need help on a question regarding thermal physics!

An ideal gas expands isothermally from a state X to a new state of volume V. The work done by the gas is W. Which of the following is correct for an adiabatic expansion of the gas from state X to a new state of volume V?

Change in internal energy - work done

A. Delta U>0 - greater than W

B. Delta U<0 - greater than W

C. Delta U>0 - less than W

D. Delta U<0 - less than W

Since transfer of thermal energy = change in internal enetgy + work done by system, i.e Q= U +W . In an Adiabatic expansion Q=0( see definition), so U =-W whcih is negative so <0, and Work done un an adiabatic expansion will be less as it is the area underneath the curve, which curves below the isothermal one

Edited by SYL
Link to post
Share on other sites

  • 2 years later...

Join the conversation

You can post now and register later. If you have an account, sign in now to post with your account.

Guest
Reply to this topic...

×   Pasted as rich text.   Paste as plain text instead

  Only 75 emoji are allowed.

×   Your link has been automatically embedded.   Display as a link instead

×   Your previous content has been restored.   Clear editor

×   You cannot paste images directly. Upload or insert images from URL.

×
×
  • Create New...